summaryrefslogtreecommitdiff
path: root/Master/texmf-dist/doc/latex/xsim/examples/xsim.texsx-369803.tex
diff options
context:
space:
mode:
Diffstat (limited to 'Master/texmf-dist/doc/latex/xsim/examples/xsim.texsx-369803.tex')
-rw-r--r--Master/texmf-dist/doc/latex/xsim/examples/xsim.texsx-369803.tex50
1 files changed, 50 insertions, 0 deletions
diff --git a/Master/texmf-dist/doc/latex/xsim/examples/xsim.texsx-369803.tex b/Master/texmf-dist/doc/latex/xsim/examples/xsim.texsx-369803.tex
new file mode 100644
index 00000000000..58b6800f937
--- /dev/null
+++ b/Master/texmf-dist/doc/latex/xsim/examples/xsim.texsx-369803.tex
@@ -0,0 +1,50 @@
+% https://tex.stackexchange.com/q/369803
+\documentclass[a4paper,parskip=half]{scrartcl}
+\usepackage[utf8]{inputenc}
+\usepackage[ngerman]{babel}
+
+\usepackage{amsmath}
+\usepackage{amsthm}
+\usepackage{amsfonts}
+\usepackage{amssymb}
+
+\usepackage[left=2cm,right=2.5cm,top=2.5cm,bottom=2cm]{geometry}
+
+\usepackage{xsim,siunitx}
+\DeclareExerciseTagging{difficulty}
+\DeclareExerciseEnvironmentTemplate{custom}
+ {%
+ \subsection*
+ {%
+ \XSIMmixedcase{\GetExerciseName}\nobreakspace
+ \GetExerciseProperty{counter}%
+ \IfInsideSolutionF
+ {%
+ \IfExercisePropertySetT{subtitle}
+ { {\normalfont(\GetExerciseProperty{subtitle})}}%
+ }%
+ }%
+ }{}
+
+\xsimsetup{
+ exercise/name = \XSIMtranslate{question} ,
+ exercise/template=custom ,
+ solution/print=true
+}
+
+\begin{document}
+
+\begin{exercise}[ID=wdsw, subtitle = Widerstandswürfel , difficulty = 2]
+ Gegeben ist ein Würfel, wobei jede der Kanten einen Widerstand von $R =
+ \SI{1}{\ohm}$ hat.
+
+ Wie groß ist der Widerstand entlang einer Raumdiagonale?
+\end{exercise}
+\begin{solution}
+ Wir wollen den Widerstand zwischen den Punkten $X$ und $Y$ bestimmen, also
+ entlang der Raumdiagonale (siehe Abb. \ref{fig:wdsws1}). Weil die
+ Raumdiagonale eine Symmetrieachse ist, sollte das Problem symmetrisch sein,
+ und deswegen eine recht einfache Lösung haben.
+\end{solution}
+
+\end{document}